FidgetBoss 4000's 2019 Mock AMC 12B Problems/Problem 2

Revision as of 15:18, 19 November 2020 by Icematrix2 (talk | contribs) (Created page with "==Problem== <i>(fidgetboss_4000)</i> In the diagram below, <math>ABC</math> is an isosceles right triangle with a right angle at <math>B</math> and with a hypotenuse of <math>...")
(diff) ← Older revision | Latest revision (diff) | Newer revision → (diff)

Problem

(fidgetboss_4000) In the diagram below, $ABC$ is an isosceles right triangle with a right angle at $B$ and with a hypotenuse of $40\sqrt2$ units. Find the greatest integer less than or equal to the value of the radius of the quarter circle inscribed inside $\triangle ABC$.

$\textbf{(A)} 26\qquad\textbf{(B)} 27\qquad\textbf{(C)} 28\qquad\textbf{(D)} 29\qquad\textbf{(E)} 30\qquad$

Solution

The quarter circle is centered at $B$ and just touches the hypotenuse at the midpoint of the hypotenuse due to symmetry in an isosceles right triangle. The value of its radius is equal to the distance between $B$ and the midpoint of the hypotenuse, and it is well known that this is half the length of the hypotenuse, or $\frac{1}{2}\cdot 40\sqrt2=20\sqrt2$. The greatest integer less than or equal to $20\sqrt2$ is $28$, thus we pick answer $\boxed{\textbf{(C) }28}$.

See also

2019 AMC 12B (ProblemsAnswer KeyResources)
Preceded by
Problem 1
Followed by
Problem 3
1 2 3 4 5 6 7 8 9 10 11 12 13 14 15 16 17 18 19 20 21 22 23 24 25
All AMC 12 Problems and Solutions

The problems on this page are copyrighted by FidgetBoss 4000's Mock American Mathematics Competitions. AMC logo.png